darya.f

New Member
ارسال ها
182
لایک ها
114
امتیاز
0
#81
پاسخ : ماراتن نظریه ی اعداد (سطح پیشرفته)

تیکه آخر چی شد که نتیجه گرفتید 'a برابر یک هستش ؟
بقىه مقادىر رو بزارىد تو معادله ,ىه سرى مقدار برا b' مىده که چکشون کنىم جواب نمىده فقط حالت a'=1 جواب مىده
 

REZA 73

Active Member
ارسال ها
139
لایک ها
184
امتیاز
43
#82
پاسخ : ماراتن نظریه ی اعداد (سطح پیشرفته)

ثابت کنید معادله زیر در مجموعه اعداد طبیعی بیشمار جواب دارد:

 

REZA 73

Active Member
ارسال ها
139
لایک ها
184
امتیاز
43
#83
پاسخ : ماراتن نظریه ی اعداد (سطح پیشرفته)

اگر
فرد باشد خواهیم داشت :
که با توجه به زوج بودن 1+
خواهیم داشت
که با توجه به عبارت بالا میتوان نتیجه گرفت :
و چون
کوچکترین عامل اول n میباشد پس
و
که اگر حاصل را در عبارت ابتدایی جاگزین کنیم و پس از کمی ساده کردن خواهیم داشت :
که با مقایسه دو طرف این تساوی به سادگی نتیجه میشود که :
که به سادگی میتوان نتیجه گرفت که این نابرابری به طوراکید جواب ندارد و تنها حالت تساوی میتوان رخ دهد که در این حالت
و همچنین در این حالت n نمیتواند عامل اولی غیر از 2 و3 داشته باشد و به عبارتی n=6 که با n>6 در تناقض است و فرض اولیه غلط بوده و
عددی زوج است .
ببخشید. طبق این راه حل یعنی شما ثابت کردین همه اعداد کامل زوج هستند؟؟
خط چهارم واضح نیس .لطفا کمی بیشتر توضیح بدید اگه ممکنه.
 
آخرین ویرایش توسط مدیر

mahmoud20ni

New Member
ارسال ها
18
لایک ها
13
امتیاز
3
#84
پاسخ : ماراتن نظریه ی اعداد (سطح پیشرفته)

خیر ثابت کرده ام اگر یک عدد کامل شرط گفته شدن در صورت سوال یعنی زوج بودن
برایش برقرار نباشد اونوقت اون عدد کامل باید زوج باشد . (تو خط سوم راه حل نوشتم این موضوع رو واضح ) در ارتباط خط چهارم هم با توجه به این که:
و تساوی نوشته شده در خط اول (با ساده کردن p_i-1 ها از عبارت سمت چپ این تساوی برای i های بزرگتر از 2 ) خواهیم داشت :
و طبق عبارت عاد کردنی که در خط 2 ام راه حل نوشته شده خواهیم داشت :
که این نیز عبارت نوشته شده در خط چهارم را نتیجه میدهد .
 

mahmoud20ni

New Member
ارسال ها
18
لایک ها
13
امتیاز
3
#85
پاسخ : ماراتن نظریه ی اعداد (سطح پیشرفته)

خیر ثابت کرده ام اگر یک عدد کامل شرط گفته شدن در صورت سوال یعنی زوج بودن
برایش برقرار نباشد اونوقت اون عدد کامل باید زوج باشد . (تو خط سوم راه حل نوشتم این موضوع رو واضح ) در ارتباط خط چهارم هم با توجه به این که:
و تساوی نوشته شده در خط اول (با ساده کردن p_i-1 ها از عبارت سمت چپ این تساوی برای i های بزرگتر از 2 ) خواهیم داشت :
و طبق عبارت عاد کردنی که در خط 2 ام راه حل نوشته شده خواهیم داشت :
که این نیز عبارت نوشته شده در خط چهارم را نتیجه میدهد .

---- دو نوشته به هم متصل شده است ----

در ارتباط با سوالی نیز که قرار داده اید a=b=c=d=2, e=4 در مسئله صدق میکند حالا با ضرب تساوی در یک عدد توان
ام کامل میتوان بی نهایت جواب طبیعی برای معادله ساخت .
 

REZA 73

Active Member
ارسال ها
139
لایک ها
184
امتیاز
43
#86
پاسخ : ماراتن نظریه ی اعداد (سطح پیشرفته)

خیر ثابت کرده ام اگر یک عدد کامل شرط گفته شدن در صورت سوال یعنی زوج بودن
برایش برقرار نباشد اونوقت اون عدد کامل باید زوج باشد . (تو خط سوم راه حل نوشتم این موضوع رو واضح ) در ارتباط خط چهارم هم با توجه به این که:
و تساوی نوشته شده در خط اول (با ساده کردن p_i-1 ها از عبارت سمت چپ این تساوی برای i های بزرگتر از 2 ) خواهیم داشت :
و طبق عبارت عاد کردنی که در خط 2 ام راه حل نوشته شده خواهیم داشت :
که این نیز عبارت نوشته شده در خط چهارم را نتیجه میدهد .
بله درسته.
البته وقتی شما ثابت کردین که کوچکترین عامل اول باید 2 باشه راه ساده تری هم وجود داشت یعنی از این استفاده کنید عددی کامل و زوج است اگر و تنها اگر به صورت:
باشد که q و
هر دو اول باشند. که دیگه حکم بدیهی میشه.
 

mahmoud20ni

New Member
ارسال ها
18
لایک ها
13
امتیاز
3
#87
پاسخ : ماراتن نظریه ی اعداد (سطح پیشرفته)

یعنی شما ثابت کردید که عددی به شکل
وجود ندارد .(با توجه اول بودن q و
) ؟!!! بعد از طرفی عددی به این شکل لزومی ندارد کامل باشد ؟ میشه اگه منظورتون رو اشتباه فهمیدم بیشتر توضیح بدید .

---- دو نوشته به هم متصل شده است ----

سوال بعدی :تمام اعداد طبیعی k را بیابید که برای ان ها معادله ی زیر بی نهایت جواب a,b,c در اعداد طبیعی داشته باشد .
 

AHZolfaghari

Well-Known Member
ارسال ها
935
لایک ها
1,654
امتیاز
93
#88
پاسخ : ماراتن نظریه ی اعداد (سطح پیشرفته)

یعنی شما ثابت کردید که عددی به شکل
وجود ندارد .(با توجه اول بودن q و
) ؟!!! بعد از طرفی عددی به این شکل لزومی ندارد کامل باشد ؟ میشه اگه منظورتون رو اشتباه فهمیدم بیشتر توضیح بدید .

---- دو نوشته به هم متصل شده است ----

سوال بعدی :تمام اعداد طبیعی k را بیابید که برای ان ها معادله ی زیر بی نهایت جواب a,b,c در اعداد طبیعی داشته باشد .


اگه که :
یعنی

پس

که واضح هست به ازای هر k بی نهایت جواب داره . پس به ازای هر k بی نهایت جواب (a,b,c) داریم !

سوال بعد :

تمام اعداد گویا m,n,p را بیابید که :

صحیح شوند
 

darya.f

New Member
ارسال ها
182
لایک ها
114
امتیاز
0
#89
پاسخ : ماراتن نظریه ی اعداد (سطح پیشرفته)



اگه که :
یعنی

پس

که واضح هست به ازای هر k بی نهایت جواب داره . پس به ازای هر k بی نهایت جواب (a,b,c) داریم !

سوال بعد :

تمام اعداد گویا m,n,p را بیابید که :

صحیح شوند
 

AHZolfaghari

Well-Known Member
ارسال ها
935
لایک ها
1,654
امتیاز
93
#90

REZA 73

Active Member
ارسال ها
139
لایک ها
184
امتیاز
43
#92
پاسخ : ماراتن نظریه ی اعداد (سطح پیشرفته)

یعنی شما ثابت کردید که عددی به شکل
وجود ندارد .(با توجه اول بودن q و
) ؟!!! بعد از طرفی عددی به این شکل لزومی ندارد کامل باشد ؟ میشه اگه منظورتون رو اشتباه فهمیدم بیشتر توضیح بدید .

---- دو نوشته به هم متصل شده است ----

فرض کنید عددی داریم که کامله و زوج هم هست.مثله x در ضمن s فرده:

حالا طبق تعریف معلومه که
باید 1 باشه یعنی s عددی اوله و حکم ثابت میشه. اونوقت توان 2 که کوچکترین عامل هست زوجه و با فرض اولیه فرد بودن تناقض داره . به غیر از عدد 6...
 

REZA 73

Active Member
ارسال ها
139
لایک ها
184
امتیاز
43
#93
پاسخ : ماراتن نظریه ی اعداد (سطح پیشرفته)

سوال بعد :

تمام اعداد گویا m,n,p را بیابید که :

صحیح شوند[/QUOTE]

حالا
هم گویا هست پس فرض کنید برابر
باشه که a و b نسبت به هم اولا.


و این تناقضه . در نتیجه
عدد طبیعیه.حالا به راحتی نتیجه میشه که :

بقیه دیگه چک کردن سادس ...
 
آخرین ویرایش توسط مدیر

Farbod9717

New Member
ارسال ها
6
لایک ها
11
امتیاز
0
#94
پاسخ : ماراتن نظریه ی اعداد (سطح پیشرفته)

اينطور كه معلومه يه ١٥ روزي اينجا داره خاك ميخوره
حالا من يه سوال بذارم....
n عددى طبيعى است و مجموع مقسوم عليه هاى مثبت آن توانى از ٢ است ثابت كنيد تعداد اين مقسوم عليه ها هم توانى از ٢ است.
 

Dadgarnia

New Member
ارسال ها
1,350
لایک ها
1,127
امتیاز
0
#95
پاسخ : ماراتن نظریه ی اعداد (سطح پیشرفته)

اينطور كه معلومه يه ١٥ روزي اينجا داره خاك ميخوره
حالا من يه سوال بذارم....
n عددى طبيعى است و مجموع مقسوم عليه هاى مثبت آن توانى از ٢ است ثابت كنيد تعداد اين مقسوم عليه ها هم توانى از ٢ است.
فرض می کنیم
واضح است که
ها فردند. پس فرض می کنیم
که m فرد است. پس داریم(
را با p نشان می دهیم):

اما واضح است که p نمی تواند دو باشد پس j برابر با صفر است که نتیجه می دهد m برابر با یک است. پس داریم:
که حکم را نتیجه می دهد.
 

math

New Member
ارسال ها
1,129
لایک ها
1,096
امتیاز
0
#96
پاسخ : ماراتن نظریه ی اعداد (سطح پیشرفته)

فرض می کنیم
واضح است که
ها فردند. پس فرض می کنیم
که m فرد است. پس داریم(
را با p نشان می دهیم):

اما واضح است که p نمی تواند دو باشد پس j برابر با صفر است که نتیجه می دهد m برابر با یک است. پس داریم:
که حکم را نتیجه می دهد.
فکر میکنم با توجه به قوانین شما باید سوال بعدی رو بذارید . :3:
 

Dadgarnia

New Member
ارسال ها
1,350
لایک ها
1,127
امتیاز
0
#97
پاسخ : ماراتن نظریه ی اعداد (سطح پیشرفته)

فکر میکنم با توجه به قوانین شما باید سوال بعدی رو بذارید . :3:
من سابقه ي خوبي توي سوال گذاشتن ندارم يكي دو بار سوال گذاشتم تاپيك خوابيد. اميدوارم اين دفعه اينجوري نشه:
يك عدد صحيح مثبت تناوبي ناميده مي شود اگر در بين هر دو رقم متوالي آن درمبناي 10 يكي زوج و ديگري فرد باشد همه ي اعداد صحيح n را چنان پيدا كنيد كه n مضربي در بين اعداد تناوبي داشته باشد.
 

TheOverlord

New Member
ارسال ها
159
لایک ها
282
امتیاز
0
#98
پاسخ : ماراتن نظریه ی اعداد (سطح پیشرفته)

لم1)دقت کنید که اگر n عددی باشد که نسبت به 10 اول است آنگاه
به پیمانه n مرتبه ای مانند m دارد.
1)ابتدا حالتی بگیرید که n نسبت به 10 اول است. پس به ازای t=2
n|(10101...010)*99

حال فرض کنید تعداد 10 ها در عبارت بالا برابر k باشد. 99k بار 10 رو پشت سرهم بنویسید و اسم این عبارت رو r بگذارید. این عبارت بدیهتا تناوبی است و ثابت میکنیم n|r.
دقت کنید که n|(10101...010)*99=r*99/s کافیست بگوییم s بر 99 بخش پذیر است در این صورت اگر s/99=l آنگاه n|r/l پس n|r اما این نیز طبق لم دو خط بدیهی است(متاسفانه از تایپ این موضوع عاجزم) پس در این حالت مساله حل است.

2)اگر ب. م. م. 100 با n برابر با 10 هم باشد حکم بدیهی است زیرا اثبات کردیم n/10 مضربی تناوبی دارد، و اگر دقت کنید رقم آخر این مضربی که ساختیم صفر است پس این مضرب هم بر n/10 و هم بر 10 بخش پذیر است و چون ب.م.م. 100 با n برابر 10 است پس این دو عدد نسبت به هم اولند پس طبق لم اقلیدس آن مضرب بر n هم بخش پذیر است.

3)حال دقت کنید که اگر n بر 20 بخش پذیر باشد آنگاه بدیهتا n مضرب تناوبی ندارد زیرا رقم آخر همه مضارب n صفر است و رقم ماقبل آخر آنها هم زوج. پس دو رقم زوج متوالی دارند.

پس باید تنها حالاتی را بررسی کنیم که ب. م.م. n با 10 برابر 2 یا 5 است یا n بر 50 بخش پذیر باشد ولی بر 4 بخش پذیر نباشد.
دو حالت اول را میتوان طبق استقرا ثابت کرد که برای توان های 2 و 5 درست است و سپس از لم1 بگوییم در این حالات مضرب تناوبی وجود دارد ، و بخش پذیری n بر 50 را میتوان مانند نتیجه گرفتن پاراگراف 2 از 1 ثابت کرد که مضرب تناوبی وجود دارد پس مضرب تناوبی برای هر n که بر 20 بخش پذیر نیست وجود دارد.
 

TheOverlord

New Member
ارسال ها
159
لایک ها
282
امتیاز
0
#99
پاسخ : ماراتن نظریه ی اعداد (سطح پیشرفته)

ثابت كنيد كه تابع حسابي
يعني تعداد اعداد اول كوچكتر يا مساوي
به فرم حاصل تقسيم دو چند جمله اي قابل بيان نيست​
 

aras2213

New Member
ارسال ها
216
لایک ها
228
امتیاز
0
پاسخ : ماراتن نظریه ی اعداد (سطح پیشرفته)

ثابت كنيد كه تابع حسابي
يعني تعداد اعداد اول كوچكتر يا مساوي
به فرم حاصل تقسيم دو چند جمله اي قابل بيان نيست​
این راه درسته؟:17:

فرض کنید که:


حالا اگه
، n امین عدد اول باشه اون وقت
اما
که تناقضه.
 
بالا